Measurement 14

You might also like

Download as pdf or txt
Download as pdf or txt
You are on page 1of 34

Measurement of Resistance

14.1 CLASSIFICATION OF RESISTANCES conditions in the circuit are not disturbed. However,
The classification of resistances, from the point of in practice this is not possible and hence both the
view of measurement, is as follows : methods give inaccurate results.
(i) Low resistances. All resistances of the order of Consider circuit of Fig. 14.1(a). In this circuit the
1 Q and under may be classified as low ammeter measures the true value of the current
resistances. through the resistance but the voltmeter does not
(ii) Medium resistances. This class includes resis­ measure the true voltage across the resistance. The
tances from 1 Q upwards to about 0.1 MQ. voltmeter indicates the sum of the voltages across the
(iii) High resistances. Resistances of the order of ammeter and the measured resistance.
0.1 MQ and upwards are classified as high
resistances.
The classification outlined above is not rigid, but
forms a basis for techniques, followed or measure­
ment, which may be different for different classes.

14.2 MEASUREMENT OF MEDIUM RESISTANCES


(«)
The different methods used for measurement of
medium resistances are : Fig. 14.1 Measurement of resistance by
ammeter-voltmeter method.
(i) Ammeter-Voltmeter method.
(ii) Substitution method. Let Ra be the resistance of the ammeter.
(iii) Wheatstone bridge method. .'. Voltage across the ammeter, Va = IRa
(iv) Ohmmeter method. Now, measured value of resistance,
_VR^a IR+1Ra (1A
14.2.1 Ammeter-Voltmeter Method R
L\n 1
— P .
-KI 1?
K a ...(14.2)
I
This method is very popular since the instruments
required for this test are usually available in the True value of resistance,
laboratory. The two types of connections employed for R=Rmt~Ra ...(14.3)
ammeter-voltmeter method are shown in Figs. 14.1(a)
and (b). In both the cases, if readings of ammeter and = R,nl
1-i
...(14.4)
Rnt 1 >
voltmeter are taken, then the measured value of
resistance is given by : Thus the measured value of resistance is higher
_ voltmeter reading _ V (14 1) than the true value. It is also clear from above that the
m ammeter reading I true value is equal to the measured only if the
ammeter resistance, Ra, is zero.
The measured value of resistance R , would be
equal to the true value, R, if the ammeter resistance is Relative error, e, = -^211—— = H4 5)
zero and the voltmeter resistance is infinite, so that the r R R "v ' ’

(421)
422 Electrical and Electronic Measurements and Instrumentation

It is clear from Eqn. 14.5 that the error in measure­ Fig. 14.1(b). If the two instruments are of 0.5%
ments would be small if the value of resistance under accuracy and are read near full scale, the instrumental
measurement is large as compared to the internal error in the result may be any thing from 0 to 1%. If
resistance of the ammeter. Therefore the circuit of read near half scale, the percentage error may be twice
Fig. 14.1(a) should be used when measuring high as great and for lower readings may mount
resistance values. considerably higher. With less accurate instruments
Consider the circuit of Fig. 14.1(b). In this circuit
the possible error, of course is increased. It is difficult
the voltmeter measures the true value of voltage but to obtain accuracy much better than 1% in a resistance
the ammeter measures the sum of currents through value under usual conditions and the error sometimes
the resistance and the voltmeter. may be considerably higher. However, the method is
useful in some laboratory work in which high
Let Rp be the resistance of the voltmeter.
accuracy is not required.
Current through the voltmeter, I = VI Iv
The suitability of a particular method for
Measured value of resistance,
resistance measurement depends upon the relative
r = V- V _ V R values of resistance under measurement and the
m2 1 Ir + Iv~V/R + V/Rv~ 1+R/Rv resistance of the meters.
The division point between the two methods is at
True value of resistance,
the resistance for which the relative errors given by
R=^n2Rv =R f----- 1------- ...(14.6) the two methods are equal.
Rv~Rm2 The relative errors for the two cases are equal
From Eqn. 14.6 it is clear that the true value of when :
resistance is equal to the measured value only if the
=— (See Eqns. 14.5 and 14.9)
resistance of voltmeter, Rv, is infinite. However, if the
resistance of voltmeter is very large as compared to
R Rv
the resistance under measurement: or when true value of resistance

Or ^>>R„i2' and therefore Rm2! Rv is very small. R = 7rX -(14.10)


For resistances greater than the value given by
We have, R = Rm z_ ...(14.7)
Eqn. 14.10 the method of Fig. 14.1(a) is used while for
lower resistances method of 14.1(b) is used.
Thus the measured value of resistance is smaller
than the true value. Example 14,1 In the measurement of a resistance, R, by
the Ammeter-voltmeter method, connections as in
R — R R^
Relative error, e = ——-----= —— ...(14.8) Figs. 14.1(a) and 14.1(b) are used. The resistance of ammeter
r R RvR is 0.01 Q and that of voltmeter, 2000 Q. In case of (b) the
The value of Rn;2 is approximately equal to R. current measured is 2 A and the voltage 180 V. Find the
percentage error in calculating resistance Ras a quotient of
er ...(14.9) the readings and the true value ofR. Also find the reading of
the voltmeter in case of (a) if the current indicated by the
It is clear from Eqn. 14.9 that the error in ammeter is 2 A.
measurement would be small if the value of resistance Solution. Case (b) See Fig. 14.1(b)
under measurement is very small as compared to the
Measured value of resistance
resistance of the voltmeter. Hence the circuit of
Fig. 14.1(b) should be used when measuring low Rm2 = V/ 1 = 180/2 =90 Q.
resistance values. Current through the voltmeter
The Ammeter-Voltmeter method, in the two forms lv = VI Rv = 180/2000 = 0.09 A.
explained above, is a simple method but is, essentially Current through the resistance
a comparatively rough method, the accuracy being
IR = I-Iv =(2.0-0.09) A.
limited by accuracy of ammeter and voltmeter used,
even if corrections are made for the voltage drop True value of resistance
across the ammeter for connections of Fig. 14.1(a) and
R = — =---- —---- = 94.24 Q.
for the shunting effect of voltmeter for connections of IR (2.0 -0.09)
Measurement of Resistance 423

Percentage error Example 14,3 A resistance R is measured using the


90 794 24=_45% connections of Fig. 14.1(a). The current measured is 10 A
= Rm2-LRx100 on range 10 A and the voltage measured is 125 V on 150 V
R 94.24
range. The scales of the ammeter and voltmeter are uniform,
Case (a) See Fig. 14.1(a)
the total scale divisions of ammeter are 100 and that of
Reading of voltmeter voltmeter are 150. The scales of these instruments are such
V = Va+VR=I(Ra + R)= 2 (0.01+ 94.24) that 1/10 of a scale division can be distinguished. The
= 188.50 V. constructional error of the ammeter is ± 0.3% and that of
voltmeter ± 0.4%. The resistance of the ammeter is 0.25 Cl.
Example 14.2 A resistance of approximate value of 80 Cl
Calculate the value of R and the limits of possible error in
is to be measured by voltmeter-ammeter method using al A
the results.
ammeter having a resistance of 2 Cl and a 50 V voltmeter
having a resistance of 5000 Q. Solution. Reading error of ammeter
(a) Suggest which one of the two methods should be = ±---- ----- x 100 = ±0.1%
used ? 10x100
(b) Supposing in the suggested method the following Reading error of voltmeter
measurements are made : = ±---- - ---- X100 = ±0.087%
I = 0.42 A and V= 35.5 V. 10x150
Total error of ammeter 81 = ±0.3 ±0.1 = ±0.4%
What is the resulting error if the accuracy of the instruments
is ± 0.5% at full scale and the errors are standard deviations. Total error of voltmeter
Solution, (a) Value of resistance for which the 8 V = ±0.4 ± 0.067 = ±0.467%
errors are equal for the two types of connections Now, resistance R = V /1 and therefore total
systematic error in measurement
R = JRaRv =V2x5000 = 100 Q
= ± 8 V ± SA = ±0.467 ±0.4 = ±0.867%
Since the resistance to be measured has a value
less than 100 Cl, the method of Fig. 14.1 (b) should be Measured value of resistance
used as it results in smaller error. Rm2 = 125/10 = 12.5 Q.
Measured value of resistance True value of resistance
Rm2 = V/ I = 35.5/0.42 = 84.52 Q i-A_
R = Rm, (See Eqn. 14.4)
True value of resistance R...,
1 ( 0 25 A
R=Rm2 = 12.5 1-— =12.25 Cl.
I 12.5 J
Therefore the value of R is specified as
84.52 --------- - --------- | = 86Q.
1-84.52/5000 J 12.25 Q ± 0.867% = 12.25 ± 0.11 Q.
(b) Error in ammeter reading 14.2.2 Substitution Method
= (0.5/100)x 1 = 0.005 A The connection diagram for this method is shown
Percentage error at 0.42 A reading in Fig. 14.2. R is the unknown resistance while S is a
= (0.005/ 0.42 )x 100= 1.19% standard variable resistance. 'A' is an ammeter and 'r'
Error in voltmeter reading is a regulating resistance. There is a switch for putting
R and S into the circuit alternately.
= 0.5 x (50/100)= 0.25 V
The switch is put at position' I' and resistance R is
.’. Percentage error at 35.5 V reading
connected in the circuit. The regulating resistance r is
= (0.25 / 35.5) x 100 =0.704% adjusted till the ammeter pointer is at a chosen scale
Since the errors correspond to standard mark. Now, the switch is thrown to position '2'
deviations, error due to ammeter and voltmeter putting the standard variable resistance S in the
= 7(1.19)2+(0.704)2 = ± 1.38% circuit. The value of S is varied till the same deflection
as was obtained with R in the circuit is obtained. The
Absolute error due to ammeter and voltmeter
settings of the dials of S are read. Since the substitution
= (1.38/100)x86«± 1.2 Q. of one resistance for another has left the current
:. The resistance is specified as 86 ± 1.2 Q. unaltered, and provided that the emf of battery and
424 Electrical and Electronic Measurements and Instrumentation

the position of r are unaltered, the two resistances Hence, unknown resistance
must be equal. Thus the value of unknown resistance
R = (S+G)-1-G
R is equal to the dial settings of resistance S. 02
= (0.5 x 106 +10 x 103)x (41 / 51) -10 x 103
= 0.4xl06Q =0.4 MQ.

14.2.3 Wheatstone Bridge


A very important device used in the measurement
of medium resistances is the Wheatstone bridge. A
Wheatstone bridge has been in use longer than almost
Fig. 14.2 Substitution method. any electrical measuring instrument. It is still an
accurate and reliable instrument and is extensively
This is a more accurate method than the used in industry. The Wheatstone bridge is an
Ammeter-Voltmeter method, as it is not subject to the instrument for making comparison measurements and
errors encountered in the latter method. However, the operates upon a null indication principle. This means
accuracy of this method is greatly affected if there is the indication is independent of the calibration of the
any change in the battery emf during the time the null indicating instrument or any of its characteristics.
readings on the two settings are taken. Thus in order For this reason, very high degrees of accuracy can be
to avoid errors on this account, a battery of ample
achieved using Wheatstone bridge. Accuracy of 0.1%
capacity should be used so that its emf remains
is quite common with a Wheatstone bridge as
constant.
opposed to accuracies of 3% to 5% with ordinary
The accuracy of the measurement naturally ohmmeter for measurement of medium resistances.
depends upon the constancy of the battery emf and of Figure 14.3 shows the basic circuit of a Wheatstone
the resistance of the circuit excluding R and S, upon bridge. It has four resistive arms, consisting of
the sensitivity of the instrument, and upon the resistances P, Q, R and S together with a source of emf
accuracy with which standard resistance S is known. (a battery) and a null detector, usually a galvanometer
This method is not widely used for simple G or other sensitive current meter. The current
resistance measurements and is used in a modified through the galvanometer depends on the potential
form for the measurement of high resistances. The difference between points c and d. The bridge is said to
substitution principle, however, is very important and be balanced when there is no current through the
finds many applications in bridge methods and in galvanometer or when the potential difference across
high frequency a.c. measurements. the galvanometer is zero. This occurs when the
Example 14.4 In a measurement of resistance by voltage from point ‘V to point ‘d equals the voltage
substitution method a standard 0.5 MD resistor is used. The from point ‘d! to point ‘If; or, by referring to the other
galvanometer has a resistance oflOkD and gives deflections battery terminal, when the voltage from point'd' to
as follows : point'd equals the voltage from point 'V to point'd.
(z) With standard resistor, 41 divisions, For bridge balance, we can write,
(zz) With unknown resistance, 51 divisions. fP=I2R ...(14.11)
Find the unknown resistance.
Solution. The deflection of the galvanometer is
directly proportional to the current passing through
the circuit and hence is inversely proportional to the
total resistance of the circuit. Let S, R and G be
respectively the resistances of standard resistor,
unknown resistor and the galvanometer. Also let 0] be
the deflection with standard resistor in circuit and 02
with unknown resistor in circuit.
£1 - R + G
02 “ S+G Fig. 14.3 Wheatstone bridge.
Measurement of Resistance 425

For the galvanometer current to be zero, the 14.2.4 Sensitivity of Wheatstone Bridge
following conditions also exist: It is frequently desirable to know the galvano­
meter response to be expected in a bridge which is
...(14.12)
1 3 P+Q slightly unbalanced so that a current flows in the
galvanometer branch of the bridge network. This may
and / -j =_L_ ...(14.13) be used for :
2 4 R+S
(z) selecting a galvanometer with which a given
where E - emf of the battery. unbalance may be observed in a specified
Combining Eqns. 14.11, 14.12 and 14.13 and bridge arrangement,
simplifying, we obtain : (zz) determining the minimum unbalance which
can be observed with a given galvanometer
in the specified bridge arrangement, and
(z'zz) determining the deflection to be expected for
from which QR = PS ...(14.15)
a given unbalance.
Equation 14.15 is the well known expression for
The sensitivity to unbalance can be computed by
the balance of Wheatstone bridge. If three of the
solving the bridge circuit for a small unbalance. The
resistances are known, the fourth may be determined
solution is approached by converting the Wheatstone
from Eqn. 14.15 and we obtain :
bridge of Fig. 14.3 to its "Thevenin Equivalent" circuit.
P
R = S± ...(14.16) Assume that the bridge is balanced when the branch
resistances are P, Q R, S so that P / Q = R / S. Suppose
where R is the unknown resistance, S is called the the resistance R is changed to R +\R creating an
'standard arm' of the bridge and P and Q are called the unbalance. This will to cause an emf e to appear across
'ratio arms'. the galvanometer branch. With galvanometer branch
In the industrial and laboratory form of the open, the voltage drop between points a and b is :
bridge, the resistors which make up P, Q and S are p =1p= _
mounted together in a box, the appropriate values ab 1 P+Q
being selected by dial switches. Battery and galvano­ E(R + AR)
Similarly, Efld=/2(R + AR) =
meter switches are also included together with a R+AR+S
galvanometer and a dry battery in the portable sets. P
Therefore voltage difference between points d and
and Q normally consist of four resistors each, the
b is :
values being 10, 100, 1000 and 10,000 Q respectively S
consists of a 4 dial or 5 dial decade arrangement of
resistors. Figure 14.4 shows the commercial form of ab R+AR+S P+Q
Wheatstone bridge. ...(14.17)
P _ R
Battery © Galvanometer terminals P+Q-R+S
terminals © ©GO
Terminals © %
R + AR P
e= E
for connecting R + AR + S ~P+S
unknown © X2 ESAR
resistance
(R + S)2 + AR(R + S)
Ratio selector
ESAR
~ (R + S)2 ...(14.18)

as AR(R + S)«(R + S)2


x 1000 x 100 x 10 x1
Let Sv be the voltage sensitivity of galvanometer.
Lock for Lock for
battery ° galvanometer0 Ohmat20°C Therefore, deflection of galvanometer is
ESAR
e = sve = sv ...(14.19)
Fig. 14.4 Commercial form of Wheatstone bridge. (R+S)2
426 Electrical and Electronic Measurements and Instrumentation

R + AR R
The bridge sensitivity SB is defined as the
deflection of the galvanometer per unit fractional R + AR + R R+R
change in unknown resistance. 'R + AR 1
Bridge sensitivity 2R + AR 2
...(14.20)
B AR/R as AR « R. ...(14.25)
_ SVESR
...(14.21) The resistance of the Thevenin equivalent circuit
“(R + S)2
is found by looking back into terminals c and d
From Eqn. 14.21, it is clear that the sensitivity of (Fig. 14.3) and replacing the battery by its internal
the bridge is dependent upon bridge voltage, bridge resistance. In most cases, however, the extremely low
parameters and the voltage sensitivity of the galvano­ resistance of the battery can be neglected and this
meter. Rearranging the terms in the expression for simplifies the solution as we can assume that
sensitivity, terminals a and b are shorted. The Thevenin equivalent
S - S»E - S» E resistance can be calculated by referring to Fig. 14.5.
B (R + S)2/SR ^+2 + 4
5 1\

...(14.22)
P +2 +Q
Q P
From Eqn. 14.22, it is apparent that maximum
sensitivity occurs where R / S = 1. As the ratio becomes
either larger or smaller, the sensitivity decreases. Since
Fig. 14.5 Finding resistance of bridge looking into
the accuracy of measurement is dependent upon terminals d and b.
sensitivity a limit can be seen to the usefulness for a
given bridge, battery and galvanometer combination. Thevenin equivalent resistance of bridge
RS PQ
For a bridge with equal arms, R = S - P = Q, R + S+ P+Q ...(14.26)
S E
Bridge sensitivity ...(14.23)
Considering AR « R
As explained above the sensitivity is maximum For a bridge with equal arms,
when the ratio is unity. The sensitivity with ratio p=Q = S = Rf
P / Q = R/ S =1000 would be about 1/250 of that for R0 = R ...(14.27)
unity ratio. The sensitivity with PIQ = R/S = 1000
The Thevenin equivalent of the bridge circuit
would similarly be about 1/250 of that for unity ratio.
therefore reduces to a Thevenin generator with an emf
Thus the sensitivity decreases considerably if the £0 and an internal resistance Rq. This circuit is shown
ratio P/ Q - R/ S is greater or smaller than unity. This in Fig- 14.6.
reduction in sensitivity is accompanied by a reduction
in accuracy with which a bridge can be balanced.
Galvanometer current. The current through the
galvanometer can be found out by finding the
Thevenin equivalent circuit. The Thevenin or open
circuit voltage appearing between terminals b and d
with galvanometer circuit open circuited is,

aa ac
£Un = E . -En = UR
2v + AR)-LP
' 1 Fig. 14.6 Thevenin equivalent circuit of
_ £(R + AR) EP Wheatstone bridge.
~ R + AR + S P+Q
The current in the galvanometer circuit
R + AR P
...(14.24) ...(14.28)
R + AR + S P+Q

For a bridge with equal arms, R = S = P = Q. where G = resistance of the galvanometer circuit.
Measurement of Resistance 427

For a bridge with equal arms, reversal. The results are obtained by averaging the
I _ E(AR/4R) two readings. This way the effect of thermo-electric
...(14.29) emfs is eliminated with the added advantage of
s (R + G)
doubling the sensitivity of the bridge.
The deflection of the galvanometer for a small
3. Temperature effects. The errors caused by
change in resistance in the unknown arm is,
change of resistance due to change of temperature
SVESAR produces serious errors in measurements especially in
0=
(R + S)2 the case of resistances made up of materials having a
Sj large value of resistance temperature coefficient. For
But Sv = example in the case of copper, which has a resistance
temperature coefficient of 0.004/qC, a change in
where S = current sensitivity of the galvanometer. temperature of 1°C will cause an error of 0.4% or 1 part
SESAR in 250.
0 =------ -l------------- j ...(14.30)
(Ro + GXR + S)2 4. Contact resistance. Serious errors may be
caused by contact resistances of switches and binding
For a bridge with equal arms,
posts. A dial may have a contact resistance of about
SEAR 0.003 Q and thus a 4 dial resistance box has a contact
0 = —l----- ----- ...(14.31)
4R(R + G) resistance of about 0.01 Q. This value is quite high
Also bridge sensitivity, especially when low resistance measurements are
being done. Another aspect of the contact resistance is
0 S. ESR
...(14.32) that error caused by it is difficult to account for since
AR/R (Rq + GXR + S)2
its magnitude i.e., magnitude of contact resistance is
For a bridge with equal arms, uncertain.
Bridge sensitivity, In precision resistance measurements, the most
accurate comparisons are made on an equal ratio
S --§£
...(14.33) bridge with a fixed standard resistance nominally
B 4(R+G)
equal to resistance under test. Then with equal leads,
equal currents and equal heating of the ratio arms, the
14.2.5 Precision Measurement of Medium
possible errors are minimized. The problem is then
Resistances with Wheatstone Bridge
reduced to that of determining the exact ratio of the
It is sometimes necessary to measure resistances
unknown resistance, R^ to the standard resistance S or
to a precision of 1 part in 10,000 or even greater by the difference between them. The different methods
comparing them with standard resistances. In such used for this purpose are :
cases more than ordinary precautions are necessary in
(/) Change in ratio arms. Small known changes are
order to secure the required accuracy. The following
made in the ratio arms and the exact balance is
factors should be taken into consideration.
obtained.
1. Resistance of connecting leads. A lead of
(n) Using a high resistance shunt. In this case the
22 SWG wire having a length of 25 cm has a resistance
bridge is balanced by means of an adjustable high
of about 0.012 Q and this represents more than 1 part
resistance put in parallel with one of the bridge arms.
in 1000 for a 10 Q resistance or more than one part in
A decade resistance box is used for this purpose.
10,000 for a 100 Q resistance.
Suppose the test resistance, R, is slightly lower in
2. Thermo-electric effects. Thermoelectric emfs value than the standard resistance, S and let the
are often present in the measuring circuit and they balance be obtained with a resistance xS, put in
must be taken into account since they affect the parallel with S.
galvanometer deflection in the same way as an emf
The resistance of this arm is then
occurring because of unbalance. The effect of
thermoelectric emfs and other parasite emfs on the xS2/(S + xS), i.e., S/(1 + 1/ x).
measurement may be eliminated by reversing the
:. Value of unknown resistance
battery connections to the bridge through a quick
acting switch and adjusting the balance until no R=£._L_.
change in galvanometer deflection can be observed on Q 1+1/x
428 Electrical and Electronic Measurements and Instrumentation

Since x is very large and therefore 1/x is very Now, for first balance,
small and hence we can write, p R + Lr + S+(L-L)r R + S+Lr ...
p — +1 — •••(0
R = ~-S(l-l/x) ...(14.34) Q S + (L-Z1)r S + (L-/1)r

also, for second balance,


For an equal arm bridge, P = Q and therefore,
P S + Lr+ R + (L-L)r S+R+Lr ....
R = S(l-l/x) ...(14.35) — +1 =-------------------- — =--------------- ...(u)
Q R + (L-l2)r R + (L-l2)r
The fractional difference, 1/x between S and R can
be known. Since this method determines the fractional From (i) and (ii), we have,
difference and x has a large value, the high resistance S + (L-Z1)r = R + (L-l2)r
put in parallel with S need not be particularly accurate.
Hence S-R^-lJr ...(14.36)
(iii) Carey foster slide wire bridge. A slide wire
bridge is used for the purpose of determining the Thus the difference between S and R is obtained
difference between the standard and the unknown from the resistance per unit length of the slide-wire
resistances. The details are explained below. together with the difference (\-l2) between the two
slide-wire lengths at balance.
14.2.6 Carey-Foster Slide-wire Bridge
The slide-wire is calibrated i.e., r is obtained by
The connections of this bridge are shown in
shunting either S or R by a known resistance and again
Fig. 14.7, a slide-wire of length L being included bet­
determining the difference in length (Z* -1'2).
ween R and S as shown. This bridge is specially suited
for the comparison of two nearly equal resistances. Suppose that S is known and that S' is its value
when shunted by a known resistance ; then
Resistances P and Q are first adjusted so that the
ratio PIQ is approximately equal to the ratio R/ S. S-R = (Z1-Z2)r and S' - R =(l\-l'2)r
Exact balance is obtained by adjustment of the sliding
S-R _ S'-R
contact on the slide-wire. Let f be the distance of the
sliding contact form the left-hand end of the slide
wire. The resistances R and S are then interchanged from which R = -^1——^1—^2 ...(14.37)
and balance again obtained. Let the distance no-w be l2. (z;-z'-z1 + z2)
Equation 14.37 shows that this method gives a
direct comparison between S and R in terms of lengths
only, the resistances of P and Q contact resistances, and
the resistances of connecting leads being eliminated.
As it is important that the two resistors R and S
shall not be handled or disturbed during the
measurement, a special switch is used to affect the
interchanging of these two resistors during the test.

14.2.7 Kelvin-Varley Slide


A Kelvin-Varley slide is used for voltage division.
This method is very precise and finds extensive
applications. A Kelvin-Varley slide is shown in
Fig. 14.7 Carey-Foster Slide wire bridge. Fig. 14.8. It consists of several decades of resistors
which are interconnected. The voltage division is
Then for the first balance carried out successively. Each voltage division decade
P _ R + fr is made up of eleven equal resistors with successive
Q~ S + (L-f)r division decades having a total resistance equal to
twice the value of a unit resistor in the previous
where r is the resistance per unit length of the decade. For example, in the Kelvin Varley Slide shown
slide-wire. in Fig. 14.8, there are four decade dividers. This
„ - ... P S+Lr
For the second balance, — =------- -— decade is constructed using 11 resistance coils having
Q R + (L-l2) a resistance of 10 kfl each. The second decade divider
Measurement of Resistance 429

has 11 resistors of 2 kQ) each


.
* Similarly, the 3rd switch contact resistance due to current sharing within
decade has 11 resistors of 400 Q each and the fourth the device. The disadvantages of the Kelvin-Varley
and final decade has 10 resistors of 80 Q each. Slide are its calibration and the errors on account of
temperature. The errors on account of temperature are
Decade 1 Decade 2 Decade 3 Decade 4
o-------- a ■ ■ o —0 ---------•--- 0 dut to the fact that the resistors carry different
Kn/10 ^1/10 ^2/10 ^3/10 1 accounts and the changes in the value of r ohm
resistance is different on account of different self­
^in/10 ^1/10 ^2/10 V3/IO 1 heating conditions. The temperature effects can be
reduced to negligible proportions by using resistors
Vin/10 ^1/10 Vz/10 ^3/10 1 made of materials having a low resistance
“0^3 temperature co-efficient. The error may be reduced to
^in/10 ^1/10 ^2/10 V3/10 u as low a value as ± 0.1 ppm. The principle of
Kelvin-Varley Slides is used with advantage in
^in/10 ^1/10
~7 potentio- meters and universal shunts.
^2/10 V3/10 1

—0 6 Figure 14.9 shows the use of Kelvin-Varley slide


Kn/10 -O । ^1/10 V2/IO ^3/10 1 in a Wheatstone bridge. The device is used to replace
vin —05 v n * the simple slide wire with the advantage that it gives
^1/10
VinA0 V2/IO
v3/10 1 greater accuracy.
~°4 J—°3
—0 4
For the case shown in Fig. 14.9
^1/10
Vin/W Kz/10 V3/10 I

J-02
R _ 7554
—03 -<v2 —03
^1/10
S ~ 5346
Vm/10 ^2/10 v3/io n vouf

—o 2
°2 —0 2 i—01
^1/10
Vin/W V2/IO ^3/10 ||

—o 1 ---O1 —0 1 • 0u
Kn/10 ^1/10 Vz/lO

—o 0 u —0 0
•------------- ------------------- i

Fig. 14.8 Kelvin-Varley divider set to 0.4274.

The use of 11 resistors to obtain a decade voltage


division enables the Kelvin-Varley Slide to have a
constant input resistance irrespective of which switch
positions are connected on the various decades.
(However this is strictly true if output terminals are
open circuited). For example decade 3 has a constant
input resistance of :
9x 400 + 1x400 + 10x80 = 4000 Q = 4 kQ.
800 + 800
Decade 2 has an input resistance of
Fig. 14.9 Kelvin-Varley slide.
9 x 2000 + 1x2000x4000 = 20,000 Q = 20 kQ.
4000 + 4000 14.2.8 Limitations of Wheatstone Bridge
The input terminal impedance is 100 kQ The use of Wheatstone bridge is limited to the
irrespective of the switch position if it is assumed that measurement of resistances ranging from a few ohm
the output current is negligible. to several megohms. The upper limit is set by the
The advantage of using the Kelvin-Varley Slide is reduction in sensitivity to unbalance caused by high
the reduction of errors which arise on account of resistance values. The upper limit can be extended to a

* Resistance of each unit in first decade is 10 kQ The second decade has a total resistance of 2 x 10 = 20 kQ. Therefore, each unit
in the second decade has a resistance of 20/10 = 2 kQ In other words, we can say that if the first decade consists of 11 coils of
resistance r ohm each, the next decade has 11 coils of r/5 ohm each and the third decade has 11 coils of 2r/5 ohm each.
430 Electrical and Electronic Measurements and Instrumentation

certain extent by increasing the emf applied to the Q = 100 £1, R = 2,005 Q and S = 200 Q. The battery has an
bridge but in this case care has to be taken to avoid emf of 5 V and negligible internal resistance. The
overheating of any arm of the bridge. Inaccuracy may galvanometer has a current sensitivity of 10 mm/pA and an
also be introduced on account of leakage over internal resistance of 100 Q. Calculate the deflection of
insulation of the bridge arms when measuring very galvanometer and the sensitivity of the bridge in terms of
high resistances. For measurement of very high deflection per unit change in resistance.
resistances, a megohm bridge (described on page 441) Solution. Resistance of unknown resistor
is preferable. required for balance
The lower limit for measurement is set by the R=(P/Q)S = (1000 /100) x 200 = 2000 Q.
resistance of the connecting leads and by contact In the actual bridge the unknown resistor has a
resistance at the binding posts. The error caused by value of 2005 Q or the deviation from the balance
leads may be corrected fairly well, but contact resis­
conditions is A R =2005 -2000 = 5 Q.
tance presents a source of uncertainty that is difficult
Thevenin source generator emf
to overcome. The lower limit for accurate measure­
ment is in the neighbourhood of 1 to 5 ohm. For low = -
-------- P
resistance measurements, therefore, a Kelvin bridge R + S P+Q
(described in Art. 14.3.2 on page 434) is generally
r 2005 1000
preferred. —5
L2005+200 1000+100
Example 14.5 A resistance of approximately 3000 Q is
= 1.0307 xlO’3 V.
needed to balance a bridge. It is obtained on a 5 dial
resistance box having steps of 1000, 100, 10, 1 and 0.1 Q. Internal resistance of bridge looking into
The measurement is to be guaranteed to 0.1 percent. For this terminals b and d.
accuracy, how many of these dials would it be worth RS PQ
adjusting ? R + S +P+Q
Solution. Limiting value of 3000 Q resistor 2005x200 + 1000x100 _ 272 8 q
= 3000 ± 3000 x (0.1 /100) = 3000 ± 3Q 2005 + 200 1000 + 100 ~
= 2997 to 3003 Q Hence the current through the galvanometer
Thus it would be worth adjusting the 1000,100, 10 1.0307 x IQ"3
A = 2.77 p A.
and 1 Q dials. R0 + g 272.8 + 100

Example 14.6 Each of the ratio arms of a laboratory type Deflection of the galvanometer
Wheatstone bridge has guaranteed accuracy of ± 0.05%, 0 = SJg =10x2.77 =27.7 mm/fi.
while the standard arm has a guaranteed accuracy of Sensitivity of bridge
± 0.1%. The ratio arms are both set at 1000 Q and the bridge
r 0 27.7 c
is balanced with standard arm adjusted to 3,154 Q. S„ ----- =------ = 5.54 mm/Q.
B AR 5
Determine the upper and the lower limits of the unknown
resistance, based upon the guaranteed accuracies of the Example 14.8 A Wheatstone bridge has ratio arms of
known bridge arms. 1000 £1 and 100 £1 and is being used to measure an
Solution. Value of unknown resistance unknown resistance of 25 £1. Two galvanometers are
available. Galvanometer 'A' has a resistance of 50 £1 and a
R = (p j Q)x S = (1000 / 1000)x 3154
sensitivity of 200 mm/pA and galvanometer 'Bt has values
= 31540. of 600 £1 and 500 mm/pA. Which of the two galvanometers
:. Percentage error in determination of R. more is sensitive to a small unbalance on the above bridge,
5R 8P , 5Q 8S and what is the ratio of sensitivities ? The galvanometer is
— = ± — ± —-- ± —
R P Q S connected from the junction of the ratio arms to the opposite
comers. Comment upon the results.
= ±0j05 ± 0j05 ± 0j01 = ± 0.2 %.
Solution. The arrangement of this bridge is
Limiting values of
shown in Fig. 14.3. Value of standard resistance under
R = 3154 ± 0.2% = 3091 to 3217 Q. balance conditions
Example 14.7 In the Wheatstone bridge of Fig. 14.3, the S = R.® =25x —°°=250Q.
values of resistances of various arms are P=1000£l, P 100
Measurement of Resistance 431

Internal resistance of bridge looking into Example 14.10 A regular Wheatstone bridge is used to
terminals b and d. measure high resistances (in the megohm range). The bridge
has ratio arms of 10,000 Q and 10 Q.
p KS PQ
R+S P+Q The adjustable arm has a maximum value of 10,000 Q. A
battery of 10 V emf and negligible resistance is connected
25x250 100x1000
=---------- +-------------- = 113.6 Q. from the junction of ratio aims to the opposite corner ?
25 + 250 100 + 1000
(a) What is the maximum resistance that can be
From Eqn. 14.30, the deflection for a small change measured by this arrangement ?
in resistance (b) If the galvanometer has a sensitivity of
_ S. ESAR 200 mm/pA and a resistance of 50 Q, how much
“ (R,, + G)(R + S)2 unbalance is needed to give a galvanometer
deflection ofl mm for the maximum resistance of
Ratio of deflections for two galvanometers part (a) ?
0^= (Sl)4ESAR / (Sl)besar (c) If the galvanometer of part (b) is replaced by a
0b (% + Gx)(R + S)2 / (Ro + Gb)(R + S)2 galvanometer of sensitivity of 1000 mm/pA and a
resistance of 1000 Q, calculate the change in
_ (^i)x ^0 + ^B
resistance to cause a deflection of 1 mm.
Solution. The arrangement of the bridge is shown
200 (113.6 + 600) „ „„ in Fig. 14.3.
=----- x ' = 1.75
500 (113.6 + 50) Now we have,
Thus galvanometer A has a sensitivity of unknown resistance, R = (P / Q) x S.
1.75 times that of galvanometer Bas far as this bridge The maximum value of
is concerned, even though on its own galvanometer A R = (maximum value of P / Q)
is less sensitive than galvanometer B. x (maximum value of S)
Example 14-9 A highly sensitive galvanometer can
detect a current as low as 0.1 nA. This galvanometer is used
in a Wheatstone bridge as a detector. The resistance of (b) Internal resistance of bridge
galvanometer is negligible. Each arm of the bridge has a
resistance ofl k£l. The input voltage applied to the bridge is
20 V. Calculate the smallest change in resistance which can
be detected. The resistance of the galvanometer can be 10xl06xl0xl03 10xl03xl0
neglected as compared with the internal resistance of bridge. ~ 10xl06 + 10xl03 10xl03 + 10
Solution. Now P = Q = R = S = lkQ = 1000Q. « 10,000 Q
It is a bridge with equal arms. From Eqn 14.30, change in resistance needed for
.’. The internal resistance of bridge 1 mm deflection
a;; etRo + GMR + S)2
Rq = R = 1000Q (See Eqn. 14.27)
SiES
Let the change in resistance be AR. 1 x (10000+ 50)(10x106 +10 xlO3)2
.'. Output voltage of bridge due to unbalance 200 x 106 x 10 x 103
Eo = EAR/4R « 0.5 MQ.
= 20x AR/(4xl000) = 5xl0“3 AR (c) The change in resistance to cause 1 mm
deflection with galvanometer having a resistance,
Current through the galvanometer
G = 1000 0
En 5x10~3AR _ __9
and current sensitivity of S(. = 1000 mm/g A is
1 x (10/100 +1000)(10 x 106 +10 x 103)2
:. The smallest change in resistance which can be ^R— 7 q
1000xl06xl0xl03
detected
AR=20x10-6Q =20 gQ. «0.1 MQ
432 Electrical and Electronic Measurements and Instrumentation

Example 14.11 A Wheatstone bridge is shown in


Fig- 14.10

Fig. 14.10 Diagram of Example 14.11.


Fig. 14.12 Wheatstone bridge of Example 14.12.
The values of resistances are :
P = lkCl, R = lk£l, S = 5 k£l, G = 100 £1. Solution. The calculated values are as under :

The Thevenin source generator voltageEQ = 24 mV and the Ra = 1200 Q,


galvanometer current is 13.6 pA. Calculate the value ofQ. Rb =(1/1600) x 1200 = 0.75 Q
Solution. The Thevenin equivalent circuit of the R2 = 800 Rb = 800 x 0.75 = 600 Q
bridge is shown in Fig 14.11. R2=Ri/1.25 = 600/1.25 = 480 Q
R,3 =0.5R,b =0.5 x 0.75 = 0.375Q.
Delta-star transformation has to be used for the
analysis.
This is shown in Fig. 14.13.

P _____RR.
« p
1200x0.75 900
—_________________ =-----------------------
Fig. 14.11 Thevenin source generator of 10 RCl + R.+Rr 1200 + 075+ R,E 1200.75 + R,L
U C
Example 14.11.
R.R. 0.75 R, 0.75 R,
= resistance of circuit looking into terminals Ra +Rbh + RrL 1200 + 0.75+ R r 1200.75+R,
l
d & c with terminals a &ab short-circuited.
RS PQ 1x5 lxQ tj
R R,L______
_ _________ Cl
1200 R, 1200 R,
__________________ ________ —______________ —____
" R + S+P+Q’l + 5 + l + Q 30 " Ra + R.+R, 1200 + 0.75+ R,g 1200.75 + R.g
b L

= 0.833 + —— kQ.
1 +Q
Now, Rq + G = 24xlO-3/13.6xl06 = 1.765 kQ

Ro =1765 -100 = 1665 Q = 1.665 kQ.


0.833 + -Q- = 1.665
1 +Q
Q = 4.95 kQ.

Example 14
*12 A modified form of Wheatstone bridge is
shown in Fig 14.12. Calculate the value of unknown
resistance, Rx, if
r=1200Q, Ra=1600Rb,
/tj =800Rfc), R2 = 1.25 R^ and R3=0.5Rb Fig. 14.13 Delta-star transformation for
are the resistance values under balanced conditions. analysis of Fig- 14.12.
Measurement of Resistance 433

The modified circuit is shown in Fig- 14.14. Current through the series resistor
= 2x50xl0-3 =100xl0-3 A.
Resistance of series resistor
R=- = 100 Q.
5 100 xlO-3
Example 14.14 In a Carey-Foster's bridge a resistance
of 1.0125 Q is compared with a standard resistance of
1.0000 Q, the slide wire has a resistance of 0.250 Q in
100 divisions. The ratio arms nominally each 10 Q, are
actually 10.05 and 9.95 Q respectively.
How far (in scale divisions) are the balance positions from
those which would obtain of ratio arms were true to their
Fig. 14.14 Configuration of bridge of Fig. 14.12 nominal value ? The slide wire is 100 cm long.
after Delta-star transformation. Solution. Balance with ratio arms equal to nominal
value. Let l^ be the distance of balance point on slide
For balance,
wire from the unknown resistance end in cm of slide
wire. Let rbe the resistance per cm length of slide wire.
*1 0.0250
r~ 100
1200 R. D 0-75 Rl
480 +------------ — Rr +----------- \ = 0.00025 Q/cm as length of wire is 100 cm.
1200.75 + Rl x 1200.75 + Rl
or In this case
600 0.375 ~
P = Q = 10Q, S = 1.0000 Q and R = 1.0125 Q.
or R = 0.3 Q.
Under balance conditions :
Example 14.13 A Wheatstone bridge is used for P_ R + Z/
measuring the value of change of resistance of a strain gauge Q~ S + (100-ZJr
which forms one of the arms of the bridge. All the arms of the
bridge including the strain gauge have a resistance of 100 Q 10 1.0125+ 0.00025L ,
or 1----- or Z. = 25 cm.
— =--------------------------- *
each. The maximum allowable power dissipation from the 10 1.0000 + 0.025-0.00025 \ 1
strain gauge is 250 mW. Determine the value of maximum
Thus the balance is obtained at 25 and 75 scale
permissible current through the strain gauge and maximum
divisions.
allowable value of bridge supply voltage. Suppose a source of
Balance arms equal to true values :
20 V is available, find the value of series resistance to be
connected between the source and the bridge to limit the Now in this case, P = 9.05 Q and Q = 10.05 Q.
input voltage of the bridge to permissible level. Under balance conditions :
Solution. The resistance of strain gauge R = 100 Q 9.95 _ 1.0125 + 0.00025 f
10.05 ~ 1.0000 + 0.025 -0JD00025ZJ
Suppose I is the current through each arm under
balanced conditions. 1-0.005 10125 + 0.00025 Z.
or -----------=---------------------- -
.-. I2R = P, 1 + 0.005 10250-0.00025/j

where P = power dissipation. or ZT = 25 cm.

Hence, maximum permisible current Thus the balance is obtained at 5 and 95 cm.

,____ ^SflxtO-3 14.3 MEASUREMENT OF LOW RESISTANCE


I = fP/R= -=0.05 A =50 mA.
V V 100 The methods used for measurement of medium
resistances are unsuitable for measurement of low
The maximum allowable voltage which can be
resistances i.e., resistances having a value under IQ.
applied to the bridge
The reason is that the resistance of leads and contacts,
= 2 x 50 x 10"3 x 100 =10 V. though small, are appreciable in comparison in the
:. Voltage across the series resistor case of low resistances. For example, a contact
= 20-10 =10V. resistance of 0.002 Q causes a negligible error when a
434 Electrical and Electronic Measurements and Instrumentation

resistance of 100 Q is being measured but the same 14.3.2 Kelvin Double Bridge Method of
contact resistance would cause an error of 10% if a low Measurement of Low Resistances
resistance of the value of 0.02 Q is measured. Hence The Kelvin bridge is a modification of the
special type of construction and techniques have to be Wheatstone bridge and provides greatly increased
used for the measurement of low resistances in order accuracy in measurement of low value resistances. An
to avoid serious errors occurring on account of the understanding of the Kelvin bridge arrangement may
factors mentioned above. PV be obtained by a study of the difficulties that arise
Low resistances are constructed with four in a Wheatstone bridge on account of the resistance of
terminals as shown in Fig. 14.15. One pair of terminals the leads and the contact resistances while measuring
CC' (called the current terminals) is used to lead low valued resistors.
current to and from the resistor. The voltage drop is Consider the bridge circuit shown in Fig. 14.15
measured between the other two terminals PF, called where r represents the resistance of the lead that
the potential terminals. connects the unknown resistance R to standard
resistance S. Two galvanometer connections indicated
by dotted lines, are possible. The connection may he
either to point '»/ or to point ‘F. When the galvano­
meter is connected to point in, the resistance, r, of the
connecting leads is added to the standard resistance, S,
resulting in indication of too low an indication for
unknown resistance R. When the connection is made
to point n, the resistance, r, is added to the unknown
resistance resulting in indication of too high a value
for R.

Fig. 14.15 Ammeter voltmeter method for Suppose that instead of using point in, which
measuring 4 terminal resistances. gives a low result, or n, which makes the result high,
we, make the galvanometer connection to any
The voltage V, indicated in Fig. 14.15, is thus 1R intermediate point 'd' as shown by full line in
times the resistance R between terminals PF and does Fig. 14.16. If at point 'd' the resistance r is divided into
not include any contact resistance drop that may be two parts, r, and such that
present at the current terminals CC'. r, P
-=- ...(14.36)
Resistors of low values are thus measured in
terms of resistance, between potential terminals,
which becomes perfectly and precisely definite in
value and is independent of the contact resistance
drop at the current terminals. Contact resistance drop
at the potential terminals need not be a source of error,
as current crossing at these terminals is usually
extremely small or even zero for null methods. Also
this contact resistance now becomes a part of the
potential circuit and is, therefore, a negligible part of
the total resistance of the potential circuit since
potential circuits have a high value of resistance.

14.3.1 Methods for Measurement of


Low Resistance
The methods for measurement of low resistance are:
Fig. 14.16 Illustrating principle of Kelvin's bridge.
1. Ammeter voltmeter method. (This method
has been explained in Art. 14.2.1 on page 421) Then the presence of r, the resistance of connec­
2. Kelvin's double bridge method. ting leads, causes no error in the result. We have,
3. Potentiometer method (This is explained in R + ri =-A-(s+r2) but -=-£ ...(14.37)
Chapter 15). r2 Q
Measurement of Resistance 435

ri = P The ratio p/q is made equal to P/Q. Under


7i + r2 P+Q balance conditions there is no current through the
galvanometer, which means that the voltage drop
between a and b, Eab is equal to the voltage drop Eanid
between a and c.
as n + n,=r and r, = ---- • r
2 P+Q Now p—
E, =— +—
Q EI
We can write Eqn. 14.37 as
and Eac.= I R + S + ...(14.39)
P\ e Q
— S 4"------- r
Ql p+Q .
Ri P (P-M)r
and ^amd “ I
or ...(14.38) p + q p + q+ r]

Therefore we conclude that making the ...(14.40)


galvanometer connection as at c, the resistance of leads
does not affect the result. For zero galvanometer deflection,
The process described above is obviously not a Fab = lE~‘amd
practical way of achieving the desired result, as there
would certainly be a trouble in determining the correct P r r, (P+dV pr
or I R+S+--
point for galvanometer connections. It does, however, P+Q p+ q+ r
suggest the simple modification, that two actual
resistance units of correct ratio be connected between ...(14.41)
points m and n, the galvanometer be connected to the
junction of the resistors. This is the actual Kelvin Now, if PI Q = pl q, Eqn. 14.41 becomes,
bridge arrangement, which is shown in Fig. 14.17. p
R = -^s- ...(14.42)
The Kelvin double bridge incorporates the idea of
a second set of ratio arms - hence the name double
Equation 14.42 is the usual working equation for
bridge - and the use of four terminal resistors for low
the Kelvin bridge. It indicates that the resistance of
resistance arms. Figure 14.17 shows the schematic
connecting lead, r, has no effect on the measurement,
diagram of the Kelvin bridge. The first of ratio arms is
provided that the two sets of ratio arms have equal
P and Q. The second set of ratio arms, pand q is used to
ratios. Equation 14.41 is useful, however, as it shows
connect the galvanometer to a point d at the appro­
the error that is introduced in case the ratios are not
priate potential between points m and n to eliminate
exactly Q equal. It indicates that it is desirable to keep r
the effect of connecting lead of resistance r between
as small as possible in order to minimize the errors in
the known resistance, R, and the standard resistance, S.
case there is a difference between ratios PIQ and p I q.
The effect of thermo-electric emfs can be
eliminated by making another measurement with the
battery connections reversed. The true value of R
being the mean of the two readings.
In a typical Kelvin bridge, the range of resistance
covered is 0.1 pQ to 1.0 Q.
The accuracies are as under :
From 1000 pQ to 1.0 Q : 0.05%.
From 100 pQ to 1000 pQ : 0.2% to 0.05%.
From 10 pQ to 100 pQ : 0.5%. to 0.2,
limited by thermoelectric emfs.
In this bridge there are four internal resistance
standards of 1 Q, 0.1 Q, 0..01 Q and 0.001 Q respec­
Fig. 14.17 Kelvin double bridge tively.
436 Electrical and Electronic Measurements and Instrumentation

14.3.3 Kelvin Bridge Ohmmeter The circuit for the Thevenin equivalent resistance
as seen from galvanometer terminals c and d is shown
This is a modified form of the Kelvin bridge and is
in Fig. 14.19(a) where Rb is the resistance in the battery
intended for the rapid measurement of the winding
resistances of machines and transformers, and for the circuit.
measurement of contact and earth conductor resistances.
The accuracy is of the order of ± 0.2%. This instrument
is direct reading and the balance is obtained by rotating
a single dial.
Figure 14.18 gives the circuit diagram of a typical
Kelvin bridge ohmmeter. The ratio arms P/Q and p/q
of Fig. 14.17 are replaced by a combination of fixed
resistors P and p and a double slide wire enabling the
bridge ratio to be varied continuously between values

Fig. 14.19 Equivalent circuit for calculation of


resistance of Kelvin bridge.

However, R,S, r and Rb, are usually very small


and may be neglected with no appreciable-loss, in
accuracy. The equivalent circuit then reduces to the
one as shown in Fig. 14.19(b). Therefore, Thevenin
equivalent resistance
PQ ! M ...(14.45)
P+Q p+q
The galvanometer current
I = —?2-
...(14.46)
s R_ 4-C
Fig. 14.18 Kelvin bridge ohmmeter.
Example 14.13 A 4 terminal resistor of approximately
Six standard resistors are used and these range
50 nQ resistance was measured by means of a Kelvin bridge
from 0.01 Q to 1000 Q. The ratio dial is calibrated from
having the following component resistances :
0.5 Q to 10.5 Q on the x 1 range using 100 Q standard
Standard resistor -100.03 Cl; Inner ratio arms = 100.31 Cl
resistor. The over all range of the instrument is from
and 200 Cl ; outer ratio arms = 100.24 Cl and 200 Cl ;
0.00005 Q to 105 Q.
resistance of link connecting the standard and the unknown
14.3.3 Unbalanced Kelvin Bridge resistance - 700 gQ. Calculate the unknown resistance to
The galvanometer current of unbalanced Kelvin the nearest 0.01 |iQ.
bridge can be found by the same Thevenin method as Solution. From Eqn. 14.41 the unknown resistance
was used for Wheatstone bridge. The Thevenin source
generator voltage, referring to Fig. 14.17 is : r=Z.s+_J!_Z_£
E0 = E - cFamd Q p+q+r\_Q p
C
p 10024
where £^=77q’£- -(14-43) x 100.03 x IO-6
200
r + -^- 200 x 700 x IQ"6 100.24 100.31
100.31+ 200 +700 x 10~6 200 200
Eamd (p+ q)r E“b -(14.44)

p+q+ r = 49.97 xl0-6fi =49.97 pQ.


Measurement of Resistance 437

Example 14.16 A Kelvin double bridge (shown in R =_PQ_ + _££


Fig. 14.17) each of the ratio arms P = Q = p = q = 1000 Q. The 0 P+Q p+q
emf of the battery is 100 V and a resistance of 5 Qis included _ loooxiooo + loooxiooo = 100Q Q
in the battery circuit. The galvanometer has a resistance of " 1000 + 1000 + 1000 + 1000 “
500 Q and the resistance of the link connecting the Galvanometer current
unknown resistance to the standard resistance may be
{ Eo -Q.OlxlQ-3
neglected. The bridge is balanced when the standard
8 ~ Ro + G " 1000 + 500
resistance S = 0.001 Q.
(a) Determine the value of unknown resistance. = 0.0067 x 10~6 A = 0.0067 pA
(b) Determine the current (approximate value) .'. Deflection of galvanometer
through the unknown resistance R at balance. 0 = $./ =200x0.0067 =1.34 mm.
(c) Determine the deflection of the galvanometer
when the unknown resistance, R, is changed by 14.4 MEASUREMENT OF HIGH RESISTANCE
0.1 percent from its value at balance. The High resistance of the order of hundreds or
galvanometer has a sensitivity of 200 mm/pA. thousands of megohm are often encountered in
Solution, (a) At balance, the value of unknown electrical equipment, and frequently must be
resistance measured.
P 1000 Common examples are:
R = —• S = x 0.001 = 0.001 Q.
Q 1000 (z) Insulation resistance of components and
(b) If we examine the Kelvin bridge circuit, we built up electrical equipment like machines
find the resistors P, Q and p, q are in parallel with the and cables ;
resistance of link, r. Since r is negligible and P, Q p and (zz) Resistance of high resistance circuit elements
q have large values, the effect of ratios arms can be like in vacuum tube circuits ;
neglected for the purpose of calculation of current. (iii) Leakage resistance of capacitors ;
.’. Current under balance conditions (iv) Volume resistivity of a material, i.e., the
resistance between two faces of unit area
Rb + R+S 5 + 0.01 + 0.0001 separated by unit distance with all
conduction from face to face being through
where Rb = resistance in the battery circuit.
the body of the material;
(c) The value R is changed by 0.1 per cent.
(v) Surface resistivity, i.e., the resistance
.-. New value of R =1.001 x 0.001 = 0.001001 Q. between two lines of unit length and unit
Voltage between points a and c (Fig. 14.17) distance apart, the lines being on the surface
_ R+ S+r „ R+S _ of the material and all conduction being on
E . =------------------- E =---------------E as r = 0 the surface.
ad R.+Rb
+ S+r R.+R
b
+ S

0.001 + 0.001001 xl00= 40x10-3 14.4.1 Difficulties in Measurement of


High Resistances
R„ 5
High accuracy is rarely required in such
From Eqns. 14.43 and 14.44, measurements, hence simple circuits are used. Since
E
0
- Eab — ^amd
F the resistances under measurement have high values,
very small currents are encountered in the measure­
ment circuits. This aspect leads to several difficulties :
<P+Q R + SJ ac
(z) The insulation resistance of the resistor may be
1000 _ 0-001001 x 40 x 10-3 comparable with the actual value of the resistor. Thus
1000 + 1000 0.001+ 0.001001 J leakage currents are produced. These leakage currents
are of comparable magnitude to the current being
= 0.01 x 10 3 V = 0.01 mV. (considering r - 0).
measured and must be eliminated from the measure­
Since R, S, r and Rfc are quite small as compared to ment. Leakage currents no doubt introduce errors, but
P,Q,p and q, we can use circuit of Fig. 14.19(b). For the they generally vary from day to day, depending upon
calculation of internal resistance, Rq as viewed from the humidity conditions and therefore cause
terminals d and b. additional unpredictable complications.
438 Electrical and Electronic Measurements and Instrumentation

(n) Due to electrostatic effect, stray changes can 14.4.2 Use of Guard Circuit
appear in the measuring circuit causing errors. Some form of guard circuits are generally used to
Alternating fields can also effect the measurements eliminate the errors caused by leakage currents over
considerably. Therefore, critical points of the insulation. Figure 14.20 illustrates the operation of a
measuring circuit must be carefully screened. guard circuit. In Fig. 14.20(a), a high resistance
(iii) In order to obtain definite ratios in the mounted on a piece of insulating material is measured
potential distribution with respect to surroundings, by the ammeter-voltmeter method. The micro­
one point of the circuit may be connected to earth for ammeter measures the sum of the current through the
accuracy in measurements. resistor (IR) and the current through the leakage path
(iv) In measurement of insulation resistance the around the resistor (IL). The measured value of
specimen often has considerable capacitance. On resistance computed from the readings indicated on
application of a direct voltage a large charging current the voltmeter and the micro-ammeter, will not be true
flows initially which gradually decays down after a value but will be in error. In Fig. 14.20(F) guard terminal
short interval. Further, insulating materials possess has been added to resistance terminal block. The
the property of dielectric absorption, i.e., after the guard terminal surrounds the resistance terminal
main charging current has decayed down, further entirely and is connected to the battery side of the
charge is slowly absorbed over a considerable period micro-ammeter. The leakage current IL, now bypasses
of time, perhaps for minutes or even hours. Thus the micro-ammeter which then indicates the current IR
measurement of true conduction current should be through the resistor and thus allows the correct
delayed until after the cessation of the charging and determination of the resistance value from the
absorbing currents. But since the absorbing currents readings of voltmeter and micro-ammeter. The guard
take a considerably long time to decay, it is usually terminal and resistance terminal are almost at the
inevitable that the conduction current measured same potential and thus there will be no flow of
includes some absorption in current. The testing current between them.
conditions, including the time between the application
of voltage and observation of the current, must be
specified.
(v) When measuring the resistance of low
conductivity conductors, insulating materials and
products, the effect of various factors upon their
resistance should be taken into account. Thus, a
change in the temperature of cardboard from 20° to
40°C is accompanied by a 13 fold change in its
resistance, changes in humidity form 10 to 60 percent
cause a 30 fold change in resistance of porcelain. Fig. 14.20 Application of guard circuit for
Besides temperature and humidity, the kind of current measurement of high resistance.
employed for measurement, the magnitude and
duration of the applied voltage, and other factors also 14.4.3 Methods for Measurement of
effect the resistance being measured. High Resistance

(vi) Fairly high voltages are used in tests in order The different methods employed are :
to raise the currents to reasonable values in order to be 1. Direct deflection method.
measured. So normally a sensitive galvanometer or 2. Loss of charge method:
micro-ammeter is required and adequate steps have to 3. Megohm bridge.
be taken to prevent damage to these delicate
4. Meggar (This is explained in Art. 9.8, page 256).
instruments.
A voltage supply of 100 V upto a few kV is often Direct deflection method. The direct deflection
used depending upon the nature and breakdown method is basically that of Fig. 14.20(F)- For high
voltage of the test object. The powe supply unit is d.c. resistances, such as insulation resistance of cables, a
transistorized source. Proper smoothing and sensitive galvanometer of 'd' Arsonval type (usually
stabilization circuits are used to ensure constancy of having a current sensitivity of at 1000 mm/pA at a
voltage with time. scale distance of 1 metre) is used in place of the
Measurement of Resistance 439

microammeter. In fact many sensitive type of replacing the insulation by a standard high resistance
galvanometers can detect currents from 0.1 — 1 nA. (usually 1 Mil), the galvanometer shunt being varied,
Therefore, with an applied voltage of 1 kV, resistances as required to give a deflection of the same order as
as high as 1012 to 10 x 1012 Q can be measured. before.
An illustration of the direct deflection method In tests on cables the galvanometer should be
used for measuring insulation resistance of a cable is short-circuited before applying the voltage. The short
shown in Fig. 14.21. The galvanometer G, measures the circuiting connection is removed only after sufficient
current IR between the conductor and the metal time has elapsed so that charging and absorption
Sheath. The leakage current IL, over the insulating currents cease to flow. The galvanometer should be
material is carried by the guard wire wound on the well shunted during the early stages of measurement,
insulation and therefore does not flow through the and it is normally desirable to include a protective
galvanometer. series resistance (of several megohm) in the
galvanometer circuit. The value of this resistance
should be subtracted from the observed resistance
value in order to determine the true resistance. A high
voltage battery of 500 V emf is required and its emf
should remain constant throughout the test.
Measurement of volume and surface
resistivity. The direct deflection method is often used
for measurement of insulation resistance of insulating
material samples available in sheet form. In such cases
we are interested in the measurement of volume
Fig. 14.21 Measurement of insulation resistance
resistivity and the surface resistivity of the material.
of cable having sheath.
Figure 14.23 shows the schematic diagram for
Cables without metal sheaths can be tested in a measurement of volume and surface resistivities of a
similar way if cable, except the end or ends on which specimen of insulating material. The specimen is
corrections are made, is immersed in water in a tank. provided with tin foil or colloidal graphite electrodes;
The water and the tank then form the return path for the upper electrode having a guard ring. For
the current. The cable is immersed in slightly saline measurement of volume resistivity (which in fact is
water for about 24 hours and the temperature is kept the specific resistance) readings of voltage applied and
constant (at about 20°C) and then the measurement is the current through the galvanometer are taken.
taken as in Fig. 14.22. Leakage currents over the edge of the specimen will
The insulation resistance of the cable R = V / ID. flow between the guard ring and the lower electrode
and hence will not introduce error into the
measurement. The volume resistivity, p, can be
calculated as follows :
Let t/j = diameter of the upper electrode d^,
r = thickness of the specimen sheet,
V1 = reading of voltmeter,
and Ij = current through galvanometer G,.

Fig.14.22 Measurement of insulation resistance of


cable having no conducting sheath.

In some cases, the deflection of the galvanometer Fig.14.23 Measurement of volume and surface resis­
is observed and its scale is afterwards calibrated by tivities of insulating material specimen.
440 Electrical and Electronic Measurements and Instrumentation

Resistance of specimen voltage V and is then allowed to discharge through


R=V1//r ButR=pf/nd12 the resistance. The terminal voltage is observed over a
considerable period of time during discharge.
.•. Volume resistivity of specimen,
The voltage across the capacitor at any instant t
nd? nd?V,
P = --- - = ----iL ...(14.47) after the application of voltage is
f tl.
V= Vexp(-f/CR)
The resistivity of a thin layer of dielectric materials
or VI v = exp(-f / CR)
is different from volume resistivity, not only because
of an adherent humidity layer but also because of or Insulation resistance
contamination, chemical alterations, absorption of t 0.4343 f
...(14.49)
gases, or structural modification. The resistance Rt ClogeV/v Clog10V/u
between two electrodes embedded in or attached to a
The variation of voltage v with time shown in
dielectric medium is composed of volume resistance
R^ and surface resistance Rs with 1 / Rf =1/ R^ + l/ Rg. Fig. 14.25.

The volume resistance, R,, can be measured


separately from surface resistance Rs by means of
guard rings as shown in Fig 14.20.
If we want to measure surface resistivity, the
galvanometer is placed in position G^. In this position
the galvanometer measures the leakage current and
current flowing between upper and lower electrodes
will be eliminated from measurement. Let
d, = diameter of lower electrode disc,
Fig. 14.25 Variation of voltage with time.
V2 = reading of voltmeter
and I2 = current through galvanometer G2. From Eqn. 14.49 it follows that if V,v,C and t are
.'. Surface resistance known the value of R can be computed.

rs = v2/i2. If the resistance R is very large the time for an


appreciable fall in voltage is very large and thus this
The leakage current flows along a path of length t process may become time-consuming. Also the
and width nd2 and therefore, surface resistivity,
voltage-time curve will thus be very flat and unless
_ Rsxnd2_nd2 V2 great care is taken in measuring voltages at the
Ps t t I2 ...(14.48)
beginning and end of the time r, a serious error may be
made in the ratio V/v causing a considerable
Other forms of specimen and electrodes are also
corresponding error in the measured value of R. More
used. For example, the electrodes and guard ring may
accurate results may be obtained by change in the
be mercury, either placed in specially machined
voltage V-v directly and calling this change as e, the
recesses, in moulded insulating materials, or retained
expression for R becomes :
by metal rings on the surface of sheet materials.
Loss of charge method. In this method,
(Fig. 14.24) the insulation resistance R to be measured
c1O8i»7l;
is connected in parallel with a capacitor C and a
electrostatic voltmeter. The capacitor is charged to
This change in voltage may be measured by a
some suitable voltage, by means of a battery having
galvanometer.
However, form the experimental point of view, it
may be advisable to determine the time t from the
discharge curve of the capacitor by plotting curve of
loge v against time t. This curve is linear as shown in
Fig. 14.26 and thus determination of time t from this
curve for the voltage to fall from V to v yields more
Flfl. 14.24 Loss of charge method. accurate results.
Measurement of Resistance 441
Megohm bridge method. Figure 14.28(a) shows a
very high resistance R with its two main terminals A
and B, and a guard terminal, which is put on the
insulation. This high resistance may be diagrama-
tically represented as in Fig. 14.28(b). The resistance R
is between main terminals A and B and the leakage
resistances RAr. and Rnr between the main terminals
A and B of from a "Three terminal resistance".

Fig. 14.26 Plot of loge v versus time.

This method is applicable to some high


resistances, but it requires a capacitor of a very high
leakage resistance as high as the resistance being
measured. The method is very attractive if the
resistance being measured is the leakage resistance of
a capacitor as in this case auxiliary R and C units are Fig. 14.28 Three Terminal Resistances.
not required.
Let us consider the hypothetical case of a 100 MQ
Actually in this method we do not measure the resistance. We assume that each of the leakage
true value of resistance since we assume here that the resistances is 100 MQ i.e., RAG = RBG = 100 MQ. Let
value of resistance of electrostatic voltmeter and the this resistance be measured by an ordinary
leakage resistance of the capacitor have infinite value. Wheatstone bridge as shown in Fig. 14.29(a). It is clear
But in practice corrections must be applied to take into that the Wheatstone bridge will measure a resistance
consideration the above two resistances. Figure 14.27 of *-°- = 67MQ instead of 100 MQ thus giving an
shows the actual circuit of the test where represents 100 + 200
the leakage resistance of capacitor. Then if R' is the error of 33 percent.
resistance of 1^ and R in parallel the discharge
equation for capacitance gives
0.4343 f
...(14.51)
Clogi0V/r

Fig. 14.27 Loss of charge method considering effects Fig. 14.29 Use of guard circuit for measurement
of leakage resistance of capacitor. of high resistances.
The test is then repeated with the unknown resis­ However if the same resistance is measured by a
tance R, disconnected and the capacitor discharging modified Wheatstone bridge as shown in Fig. 14.29(b)
through Rr The value of R1 obtained from this second with the guard connection G connected as indicated,
test and substituted into the expression the error in measurement is considerably reduced. For
RRa the arrangement shown in Fig. 14.29(b) resistance RBG
...(14.52)
R + R^ is put in parallel with the galvanometer and thus it has
no effect on the balance and only effects the sensitivity
in order to get value of R.
of the galvanometer slightly. The resistance
The leakage resistance of the voltmeter, unless Rag =100MQ is put in parallel with a resistance
very high should also be taken into consideration. P = 100 kQ and therefore for the arrangement shown
442 Electrical and Electronic Measurements and Instrumentation

the measured value has an error of only 0.01 percent Let us consider an annular ring of width dx at a
and this error is entirely negligible for measurements radius x from the centre as shown in Fig- 14.31.
of this type. Insulation resistance of this annular ring is :
The arrangement of Fig. 14.30 illustrates the
operation of a Megohm bridge.
Figure 14.30 shows the circuit of a completely
self-contained Megohm bridge which includes power
supplies, bridge a members, amplifiers, and
indicating, instrument. It has a range from 0.1 MQ to
106 MQ. The accuracy is within 3% for the lower part
of the range to possible 10% above 10,000 MQ.

Fig. 14.31 Insulation resistance of cable.

Total insulation resistance of cable

Dr2 pdX
d/2 2 nxL
p , D 2.303p D
2 tcL d 2 tiL a
0.367 , D
Fig. 14.30 Megohm bridge. = —pl0S>»7
Sensitivity for balancing against high resistance is Now in this example we have,
obtained by use of adjustable high voltage supplies of D = 25xl0-3m ; d = 5xl0~3 m ;
500 V or 1000 V and the use of a sensitive null L = 1 km = 1000 m and R = 16,000 Q.
indicating arrangement such as a high gain amplifier
Specific resistance
with an electronic voltmeter or a C.R.O. The dial on Q
is calibrated 1 -10 - 100 -1000 MQ, with main decade RL 16000x1000
p - ---------------------
1-10 occupying greater part of the dial space. Since 03671og10D/d 0367(logw2.5 /0.5)
unknown resistance R = PS/ Q, the arm Q is made, = 62.5xl0-6 Qm.
tapered, so that the dial calibration is approximately
logarithmic in the main decade, 1-10. Arm S gives Example 14.18 A test voltage is applied for several
five multipliers, 0.1, 1, 10, 100 and 1000. minutes between the conductor of a 400 metre length of
cable and earth. The galvanometer connected in series reads
The junction of ratio arms P and Q is brought on
250 divisions, the value of universal shunt being 2.5 with a
the main panel and is designated as 'Guard' terminal.
standard resistance ofl MQ in circuit, the scale reading is
Example 14^7 Derive an expression for insulation 350, the value of shunt being 1000. Calculate the insulation
resistance of single core cable. The conductor of a cable has a resistance of the cable. What would be the insulation
diameter of 5 mm and the overall diameter of the cable is resistance of the same cable of length 100 metre ?
25 mm. If the insulation resistance of the cable is 16,000
Solution. Since we have a universal shunt with
Tl/km, calculate the specific resistance of insulating material.
the galvanometer and therefore the current through
Solution. Let the circuit is proportional to the deflection of the
d = diameter of conductor, galvanometer times universal shunt multiplier,
D = diameter of insulated cable, Deflection of galvanometer with unknown resistance
L= length of cable, and ‘R’ is circuit 350 divisions and the shunt multiplier
p = resistivity of insulating material. is 2.5.
Measurement of Resistance 443

Current through the circuit with unknown Thus we have,


resistance connected is : — 0 201 = 0.667 or
R' =^=^ R'=0.667 R.
IrKx 250x2.5 = 625 K R 0.301
where K is a constant. P' ^X10 OAA7P KX10
Now R =-------- or 0.667 R=----------------
R + 10 R + 10
Deflection of galvanometer with standard
R = 5MQ
resistance 'S' is circuit is 250 divisions and the shunt
multiplier is 1000.
14.5 MEASUREMENT OF EARTH RESISTANCE
.’. Current through the circuit with standard
The provision of an earth electrode for an
resistance connected is :
electrical system is necessitated by the following
Is = K x 350 x 1000 =350000 K. reasons :
Now IR R = I S-E, 1. All the parts of electrical equipment, like
where E = voltage of the source. casings of machines, switches and circuit breakers,
lead sheathing and armouring of cables, tanks of
.'. Insulation resistance of 400 metre long cable.
transformers, etc. which have to be at earth potential,
„ Ic „ 3,50,000 K must be connected to an earth electrode. The purpose
R = — • S =------------- x 1 = 560 MQ.
IR 625 K of this is to protect the various parts of the installation,
The insulation resistance is inversely proportional as well as the persons working against damage in case
to length of cable and therefore insulation resistance of the insulation of a system fails at any point. By
1000 metre long cable connecting V these parts to an earthed electrode a
continuous low resistance path is available for leakage
= 560 x (40011000) = 224 MQ.
currents to flow to earth. This current operates the
Example 14.1g A length of cable is tested for insulation protective devices and thus the faulty circuit is
resistance by the loss of charge method. An electrostatic isolated in case a fault occurs.
voltmeter of infinite resistance is connected between the 2. The earth electrode ensures that in the event of
cable conductor and earth, forming therewith a joint
over voltage on the system due to lightning discharges
capacitance of 600 pF. It is observed that after charging the
or other system faults, those parts of equipment which
voltage falls from 250 V to 92 V in 1 minute. Calculate the
are normally dead as far as voltages are concerned, do
insulation resistance of the cable.
not attain dangerously high potentials.
Solution. From Eqn. 14.49 insulation resistance of
3. In a three phase circuit the neutral of the system
cable,
is earned in order to stabilize the potential of the
R_ 0.43431 _ 0,4343x60
circuit with respect to earth.
“ C(log10V/v) 600xl0-12(logw250/92)
An earth electrode will only be effective so long it
= 100X 109Q = 100,000 MQ. has a low resistance to the earth and can carry large
Example 14.20 A cable is tested by loss of charge currents without deteriorating. Since the amount of
method using a ballistic galvanometer, with following current which an earth electrode will carry is difficult
results : Discharged immediately after electrification, to measure, the resistance value of the earth electrode
deflection 200 divisions. Discharged after 30 s and after is taken as sufficiently reliable indication of its
electrification : (i) deflection 126 divisions, (ii) when in effectiveness. The resistance of earth electrode should
parallel with a resistance of 10MD, deflection 100 divisions. be low to give good protection and it must be
Calculate the insulation resistance of the cable. measured.
Suppose R is the insulation resistance of
Solution. The main factors on which the resistance of any
the cable and let R' be the resultant resistance of earthing system depends are :
parallel combination of insulation resistance, R and 1. Shape and material of earth electrode or
the 10 MQ resistance. electrodes used.
0.4343x30 _ 0.4343x30 2. Depth in the soil at which the electrodes are
Clog^2(jb/126 " 0.201C burned.
3. Specific resistance of soil surrounding and in
0.4343x30 _ 0.4343 x 30
and the neighbourhood of electrodes. The specific
ciog;joo7ioo ~ 0.301 C
resistance of the soil is not constant but varies from
444 Electrical and Electronic Measurements and Instrumentation

one type of soil to another. The amount of moisture


present in the soil affects its specific resistance and
hence the resistance of earth electrode is not a constant
factor but suffers seasonal variations. This calls for
periodic testing to ensure that the earthing system
remains reasonably effective.
The specific resistance of soils varies between
wide limits and is very much dependent upon its
moisture content. Approximate figures for specific
resistance of soil are 80 x 103 Qm in for moist clay to
80xl06 Qm for sand of normal moisture content. A
decrease of moisture content of 30% is capable of
producing an increase of 300% to 400% in specific
resistance. Thus it is necessary to make regular checks
for earth resistance during the year round.

14.5.1 Methods of Measuring Earth Resistance


1. Fall of potential method. Figure 14.32 shows
the circuit for measurement of earth resistance with
fall of potential method. A current is passed through
earth electrode an auxiliary electrode B (which is
usually an iron spike) inserted in earth at a distance
away from the earth electrode. A second auxiliary
electrode A is inserted in earth between E and B. The
potential difference V between E and A is measured Fig. 14.33 Distribution of potentials between two
for a given current I. The flow of ground currents is earthing electrodes.
shown in Fig. 14.33(c).
The position of electrodes E and B is fixed and the
position of electrode A is changed and resistance
measurements are done for various positions of
electrode A
A graph is plotted between earth resistance
against the distance between electrode E and A This
graph is shown in Fig. 14.34.

Fig. 14.32 Measurement of earth resistance by fall


of potential method.

The lines of the first electrode current diverge and


those of the second electrode current converge. As a
result the current density is much greater in the
Fig. 14.34 Variation of earth resistance with distance
vicinity of the electrodes than at a distance from them.
between electrode E and A.
The potential distribution between the electrodes is
shown in Fig. 14.33(b). It is apparent from this curve From Fig. 14.34, it is clear that the measured value
that the potential rises in the proximity of electrodes E of earth resistance depends upon the position of the
and B and is constant along the middle section. The auxiliary electrode A The earth resistance rises
resistance of earth, therefore, is rapidly initially. When the distance between earth
Re=V/I or VEA/I. electrode E and auxiliary electrode A is increased, it
Measurement of Resistance 445

then becomes constant, and when the auxiliary the other as the commutator rotates. The second pair
electrode A approaches the auxiliary electrode B, the of each of set of brushes is positioned on the commu­
resistance rises again. The placing of electrodes is thus tator so that continuous contact is made with one
very important and serious error may be caused by segment whatever the position of the commutator.
incorrect placing of the electrodes. The correct value of The earth tester has four terminals Pv P2. Two
resistance of earth, RE, is when the auxiliary electrode terminals P] and CT are shorted to form a common
Ais at such a distance that the resistance lies on the flat point to be connected to the earth electrode. The other
part of curve of Fig. 14.30. two terminals P2 and C2 are connected to auxiliary
The spacing between the earth electrode E and the electrodes P and C respectively.
auxiliary electrodes A, B should be large so as to get The indication of the earth tester depends upon
proper results. The distance may be a few hundred the ratio of the voltage across the pressure coil and the
metres in case the earth resistance is low. current through the coil. The deflection of its pointer
2. Earth tester. The resistance of earth can be indicates the resistance of earth directly. Although the
measured by an earth tester shown in Fig. 14.35. The "Earth Tester", which is a permanent magnet moving
"Earth Tester" is a special type of Meggar (See Art. 9.8 coil instrument and can operate on d.c. only, yet by
page 256) and it has some additional constructional including the reverser and the rectifying device it is
features additional constructional features and they possible to make measurements with a.c. flowing in
are : the soil.
(i) a rotating current reverser, and The sending of a.c. current through the soil has
(n) a rectifier many advantages and therefore this system is used.
The use of a.c. passing through the soil eliminates
unwanted effects due to production of a back emf in
the soil on account of electrolytic action. Also the
instrument is free from effects of alternating or direct
currents presents in the soil.
Example 14.21 On a 250 V supply a fault having a
resistance of 200 develops between the unearthed end of the
winding of an electric heater and the frame. If the resistance
of the substation earth electrode is 4 O that of human body
2000 0, and the safe maximum current through the body is
25 mA, what is the safe maximum resistance of consumer's
earth electrode ?
Solution. Figure 14.36 shows the diagrammatic
representation of the problem.

Fig. 14.35 Earth tester.

Both these additional features consist of simple Fig. 14.36 Diagram of Example 14.20.
commutators made up of 'L' shaped segments. They
Let R be the resistance of earth electrode at the
are mounted on the shaft of the hand driven generator.
consumer end.
Each commutator has four fixed brushes. One pair of
each set of brushes is so positioned that they make Voltage drop across the human body
contact alternately with one segment and then with = 25x10“3x2000 = 50 V.
446 Electrical and Electronic Measurements and Instrumentation

Voltage between points a and b In both cases, the loop circuit formed by the cable
= 250-50=200 V. conductors is essentially a Wheatstone bridge consis­
r 200 200 . ting of resistances P, Q, R and X. G is a galvanometer
Current I =------- =----- A. for indication of balance.
20 + 4 24
Current through human body
= I------ ---- =25xl0-3 A.
R+2000
200 R = 25xl0~3
or
24 R + 2000
or 200 R = 0.6 + 1200
or resistance of earth electrode on consumer end R « 6Q.

14.6 LOCALIZATION OF CABLE FAULTS


In this section, faults occurring in cables which are
in use on lower distribution voltages are considered.
The common faults which are likely to occur in such
cables are :
1. Ground fault. The insulation of the cable may
breakdown causing a flow of current from
the core of the cable to the lead sheath or to
the earth. This is called "Ground Fault" .
2. Short circuit fault. If the insulation between
two conductors is faulty, a current flows
between them. This is called a "Short Circuit
Fault".

14.6.1 Methods Used for Localizing Ground


and Short Circuit Faults Fig. 14.37 Murray loop test.

The methods used localizing ground and short The resistors P, Q forming the ratio arms may be
circuit faults differ from those used for localizing open decade resistance boxes or slide wires.
circuit faults.
In the case of multicore cables it is advisable, first Under balance conditions :
of all, to measure insulation resistance of each core to
earth and also between cores. This enables us to sort R P R+X P+Q
out the core that is earthed in case of ground fault ;
and to sort out the cores that are shorted in case of a X = -^(R + X) ...(14.53)
short circuit fault. Loop tests are used for location of
ground and short circuit faults. These tests can only be where (R + X) is total loop resistance formed by the
used if a sound cable runs along with the faulty cable sound cable and the faulty cable. When the conductors
or cables. The loop tests work on the principle of a have the same cross-sectional area and the same
Wheatstone bridge. The advantage of these tests is resistivity, the resistances are proportional to lengths.
that their set up is such that the resistance of fault is If I, represents the length of the fault from the test end
connected in the battery circuit and therefore does not and I the length of each cable.
effect the result. However, if the fault resistance is
high, the sensitivity is adversely affected. In this Then
section only two types of tests viz., Murray and Varley
loop tests are being described.
'^TTq'2' -<14-54)

Murray loop test. The connections for this test The above relation shows that the position of the
are shown in Fig. 14.37. Figure 14.37(a) relates to the fault may be located when the length of the cable is
ground fault and Fig. 14.37(b) relates to the short known. Also, the fault resistance does not alter the
circuit fault. balance condition because its resistance enters the
Measurement of Resistance 447

battery circuit hence affects only the sensitivity of the At balance :


bridge circuit. However, if the magnitude of the fault R+X _ P
...(14.55)
resistance is high, difficulty may be experienced in $! " Q
obtaining the balance condition on account of decrease
This determines R + X i.e., the total loop resistance
in sensitivity and hence accurate determination of the
as P, Q and S1 are known.
position of the fault may not be possible. In such a
The switch K is then thrown to position Z2Z and the
case, the resistance of the fault may be reduced by
bridge is rebalanced. Let the new value of S for balance
applying a high direct or alternating voltage - in
be S2. The four arms of the bridge now are
consistance with the insulation rating of the cable - on
P, Q, R, X + S2.
the line so as to carbonize the insulation at the point of
the fault. At balance,
R _P
Varley loop test. In this test we can determine X + S2 " Q
experimentally the total loop resistance instead of
calculating it from the known lengths of the cable and R + X + Sy P+Q
or------------ - =-------
its resistance per unit length. The necessary' X + S2 Q
connections for the ground fault are shown in
(R + X)Q~S,P
Fig. 14.38(a) and for the short circuit fault in or X = -------- —---- 2- ...(14.56)
Fig. 14.38(b). The treatment of the problem, in both P+Q
cases, is identical. Hence X is known from the known values of
P, Q, S2 from this equation and R + X (the total
resistance of 2 cables) as determined from Eqn. 14.55.
Knowing the value of X, the position of the fault is
determined.
Now
= -L or L =—— 21 ...(14.57)
21 1 R+X
where /1 = length of fault from the test end
and I - total length of conductor.
Equations 14.54 (for Murray loop test) and 14.56
(for Varley loop test) are valid only when the cable
sections are uniform throughout the loop. Corrections
must be applied in case the cross-sections of faulty and
sound cables are different or when the cross-section of
the faulty cable is not uniform over its entire length.
Since temperature affects the value of resistance,
corrections must be applied on this account if the
temperatures of the two cables are different.
Corrections may also have to be applied in case the
cables have a large number of joints.

Example 14.22 In a test for a fault to earth by Murray


loop test, the faulty cable has a length of 5.2 km. The faulty
cable is looped with a sound cable of the same length and
Fig. 14.38 Varley loop test. cross-section. The resistances of ratio arms are 100 Q and
41.2 Q at balance. Calculate the distance of the fault from
A single pole double throw switch K is used in the test end.
this circuit. Switch K is first thrown to position ZJ_Z and If the decade resistance boxes forming the ratio arms have
the resistance S is varied and balance obtained. limits of error of ± 0.5% (standard deviation) of the dial
Let the value of S for balance be Sr The four arms reading, what is the limit of error in the above calculated
of the Wheatstone bridge are P, Q, R + X, Sj. result ?
448 Electrical and Electronic Measurements and Instrumentation

Solution. Let X be the resistance of cable from the Example 14.24 A short circuit fault is located by Varley
test end to place of fault and R + X be the total loop test. The circuit of Fig. 14.38 (b) is usedfor the purpose.
resistance of loop. P and Q are the ratio arms. The ratio arms are set at P = 5£l and Q=10Q and the
X/R = Q/P values of variable resistance S are 16 Q for position 1 of
switch Kand 7Q for position 2. The sound and faulty cables
” x = 77q(X+r) are identical and have a resistance of 0.4 Cl/km. Determine
the length of each cable and the distance offault from the test
41 2
= ^(X+R) = 0.292 (X+R) end.
Solution. Let Sj be the value of resistance S2 for
Let /1 be the distance of fault from test end / be
the length of each cable and r resistance per unit position 1 and S2 for position 2.
length. R+X _ P
X = rf and X + R = 2rl S. ~Q
:. Distance of fault from test end : Hence resistance of loop
f =0.292x2x5.2 =3.03 km. P 5
R + X = —S. = —xl6 = 8Q.
Limiting fractional error of unknown resistance Q 1 10
8X_ fSP? ( 5P + 6Q^_ If ±0-5 Y f±0.5?
Resistance of each cable = 8/2=40
x " p J \ p+q J “ vvloo-) A wo~J
.'.Length of each cable = 4/0.4 = 10 km.
= 0.707%. At positron 2 we have,
Therefore limiting error of distance P _ P
= 3.03 x 0.7071100 x 1000 = 21.4 m. X+S2 " Q

Example 14.23 In a test by Murray loop method for a R + X + Sr. P+Q


or ------------ - =-------
fault to earth on a 520 metre length of cable having a X + S2 Q
resistance of 1.1 Q per 1000 metre, the faulty cable is looped
with a sound cable of the same length but having a 8 + 7 5 + 10
or -------=-------- .
resistance 2.29 Q per 1000 metre. The resistances of the X+7 10
other two arms of the testing network, at balance, are in the or X = 3.0 Q.
ratio of 2.7 : 1. Calculate the distance of fault from the
.•. Distance of fault from testing end
testing end of test cable.
= 3/0.4 = 7.5 km.
Solution. Suppose and r2 are the resistances per
unit length of faulty and sound cable respectively. Example 14.23 A Wheatstone bridge is connected for a
Varley loop test as shozvn in Fig. 14.39 when the szvitch is in
Let I be the length of each cable and f be the
position 1, the bridge is balanced with R1 =1000Cl,
distance of fault from the test end.
R2 = 2000 Q, R3 = 100 Q. When switch is in position 2, the
Referring to Fig. 14.37(a), bridge is balanced with R}=1000D, 1^=2000(71 and
X=Q
R3 = 99 Q. If the resistance of the earthed wire is 0.15 km,
R P how many metres from the bridge has the ground fault
or X= —^-(X+R) -(0 occurred ?
P+Q

and X + R = rj + r2l

1.1 ! 2.92
1000 + 1000 J x 520 = 1.76 Q.

Substituting these values in (z), we have


/
— /] = xl.76.
1000 1 2.7 + 1
f = 432 m. Fig. 14.39 Diagram of Example 14.23.
Measurement of Resistance 449

Solution. When the switch is at position 1 198 + 2 Rx =200-Rx


2000/1000 =(Ra + R{?)/100 or Rx = 0.67 Q.

or Ra + Rb=200Q Resistance of fault from bridge


When the switch is at position 2
0.15
2000 _ Ra+Rb-Rx
= 4466.7 m.
1000 99+ Rx

Review Questions

1. (a) Classify the resistances from the point of view 8. Draw the circuit of a Kelvin-Varley slide and
of measurements. explain its working and advantages.
(b) Describe in brief the different methods used for
9. What are the different problems associated with
measurement of medium resistances.
measurement of low resistances ? Explain the
2. Describe the ammeter-voltmeter method of
principle of working a Kelvin's Double Bridge and
measurement of resistances. There are two ways in
explain how the effect of contact resistance and
which the circuit of ammeter voltmeter method
resistance of leads is eliminated.
can be used
10. Draw the circuit of a Kelvin's Double Bridge used
(i) ammeter connected to the side of unknown
for measurement of low resistances. Derive the
resistance and
condition for balance.
(ii) voltmeter connected to the side of unknown
11. What are the different difficulties encountered in
resistance.
the measurement of high resistances ? Explain how
Derive the condition which decides which circuit is
these difficulties are overcome.
to be used for a particular set of ammeter,
12. Explain the loss of charge method for
voltmeter and unknown resistance. Assume equal
measurements of insulation resistance of cables.
relative error in both the cases.
13. Differentiate between volume and surface
3. Describe the substitution method of measurement
resistivities. Explain the method of measuring
of medium resistances. List the factors on which
them.
the accuracy of the method depends.
14. Describe the working of a Megohm bridge.
4. Draw the circuit of a Wheatstone bridge and derive
the conditions of balance. 15. What is the importance of the value of Earth's
5. Derive the expression for bridge sensitivity for a resistance. What are the factors which influence its
Wheatstone bridge with equal arms. Find also the value ? Describe the fall of potential method for
expression for current through the galvanometer measurement of earth resistance.
for a small unbalance. 16. Describe the construction and working of an Earth
6. What are the different factors which affect the Tester. Explain how it can be used for measure-
I

precision measurement of medium resistances ment of resistance of an earthing electrode.


with Wheatstone bridge ? Explain how their effects 17. Describe the Murray loop test for localization of
are minimized/eliminated. ground and short circuit faults in cables.
7. Describe the working of a Carey-Foster Slide-wire 18. Describe the Varley loop test for localization of
bridge. ground and short circuit faults in cables.
450 Electrical and Electronic Measurements and Instrumentation

Unsolved Problems

1. A voltmeter of resistance 500 Q and a milliammeter The galvanometer has a resistance of 100 Q, a
of 1.0 Q resistance are used to measure a resistance sensitivity of 100 mm/pA and is connected across
by ammeter voltmeter method. If the voltmeter AC. A source of 4 V d.c. is connected across BD.
reads 20 V and milliammeter 100 mA. Calculate the current through the galvanometer
and its deflection if the resistance of ami DA is
Calculate the value of measured resistance
changed from 400 Q to 401 Q.
(i) if the voltmeter is put across the resistance and
the milliammeter connected in series with the [Ans. 1.63 pA ; 16.3 mm]
unknown resistance 6. In a Wheatstone bridge, the ratio arms AB = 10 Q
(if) if the voltmeter is put across the unknown and BC = 100Q ; standard-resistance across
resistance with ammeter connected on the CD = 10 Q. The shunt across 10 Q ratio arm has to
be changed from 22310 to 27670 Q, when the
supply side.
resistor was changed for Rj in the arm DA.
[Ans. 199 Q ; 333 Q]
Calculate in magnitude the difference between the
2. In a laboratory a voltmeter of 200 Q resistance and resistances of Rj and R^. The bridge is balanced in
an ammeter of 0.02 Q resistance are available. both the cases. [Ans. 86.8 pQ]
Calculate the value of resistance that can be
measured by the ammeter voltmeter method for 7. A modified Wheatstone bridge network is
which the two different circuit measurements give constituted as follows :
equal errors. [Ans. 2 Q] AB is a resistance P in parallel with resistance p;
3. In a measurement of resistance by the substitution BC is a resistance Q in parallel with a resistance q;
method a standard resistance of 100 kQ is used. CD and DA are resistances R and S respectively.
The galvanometer has a resistance of 2000 Q, and
The nominal values of P, Q and S are each 10 Q.
gives the following deflections :
With resistance R in circuit, balance is obtained
(i) With unknown resistance : 46 divisions, with p = 30,000 Q and q = 25,000. With R replaced
(n) With standard resistance : 40 divisions. by a standard resistance of 10 Q, balance is
obtained when p = 15,000 Q and q = 40,000 Q.
Find the value of unknown resistance
Calculate the value of R. [Ans. 9.99952 Q]
[Ans. 86.7 kQ]
8. A Wheatstone bridge is used to measure the
4. The four arms of a Wheatstone bridge are as resistance of a resistor. The bridge was balanced
follows : with the values shown in Fig. 14.40. It is found that,
AB = 100Q; BC= 10Q; due to presence of chemical impurities emfs are set
CD = 4Qand DA = 50Q.
The galvanometer has a resistance of 20 Q and is
connected across BD. A source of 10 V d.c. is
connected across AC Find the current through the
galvanometer. What should be the resistance in the
arm DA for no current through the galvanometer ?
[Ans. 5.15 mA ; 40 Q]
5. The four arms of a Wheatstone bridge are as
follows :
AB= 100Q;
BC=1000Q;
CD = 4000 Q and
Fig. 14.40 Diagram of Problem 14.8.
DA = 400 Q.
Measurement of Resistance 451

up at points xx and x2 of respective values 12 mV time, s voltage, V


and 22 mV with polarities being shown in the
0 150
diagram. Calculate ;
100 121
(i) the apparent value of X,
200 97
(ii) the true of X,
300 83
(iii) the % error in the apparent value of X.
400 65
[Ans. 103 Q ; 115.7 Q ; - 12.3%]
500 57
9. A Kelvin bridge is balanced with the following
constants : 600 340

Outer ratio arm 100 Q and 1000 Q ; The further set of readings was taken with resistor
Inner arms, ratio 99.92 Q and 1000.6 Q ; R removed from circuit, as follows :

Resistance of link, 0.1 Q; time, s voltage, V


Standard resistance, 0.00377 Q. 0 150
Calculate the value of unknown resistance. 200 143

[Ans. 0.000467 Q] 400 133


10. The ratio arms of a kelvin bridge are 100 Q each. 600 121
The galvanometer has an internal resistance of 500 Q
The readings of voltmeter were subject to large
and a current sensitivity of 200 mm/pA. The
random errors. Calculate the value of resistance R.
unknown resistance is 0.1002 and the standard
resistance is set at 0.1000 Q. A d.c. current of 10 A is [Ans. 52.5 MQ]
passed through the standard and the unknown 14. A high resistance of 200 MQ has a leakage
resistance from a 2.2 V battery in series with a resistance of 400 MQ between each of its main
rheostat. terminals and the guard terminal. Find the
percentage error in measurement if the above
Calculate the deflection of the galvanometer.
resistance is measured by an ordinary Wheatstone
Neglect the resistance of the link. Find also the
bridge without providing guard circuit.
resistance unbalance to produce a deflection of 1 mm.
[Ans. 20% low]
[Ans. 3320 mm ; 0.6 pQ]
15. Two mains are working at potential difference of
11. Calculate insulation resistance of a cable in which 220 V. A 250 V voltmeter having 10,000 Q/V when
the voltage falls from 100 to 80 V in 20 s. The
connected between positive main and earth reads
capacitance is 300 pF. [Ans. 29800 MQ]
149 V and the reading is 42 V when connected
12. A cable immersed in a testing tank is charged to a
between negative main the earth. Calculate the
voltage of 200 V with an electrostatic voltmeter
insulation resistance of each main with respect to
connected between core and tank. After one
earth. [Ans. 1.73 and 0.486 MQ]
minute's electrification, the cable and voltmeter are
16. A feeder cable 250 metre long has a fault to earth.
isolated and is found that in 20 s the voltage falls to
The fault is localized by the following resistance
150 V. The test is repeated with a resistance of 20 MQ
measurements between earth and one end of cable
between core and tank and the voltage is found to
(a) distant end insulated, 6.95 Q (b) distant end
fall to 100 V in the same time. Calculate the
insulation resistance of the cable, that of the earthed, 1.71 Q. The cable has a total resistance of

voltmeter is given as 50 MQ. 1.80 Q. Find the resistance of fault and its distance
from test end.
13. The following observations were made for a loss of
17. In a Murray loop test for ground fault on a
charge method for the determination of a high
resistance R. The charged capacitor of capacitance 500 metre long cable having a resistance of
1.6 Q/km, the faulty cable is looped with a sound
12.5 pF was connected across an electrostatic
! cable of same length and cross-section. If
voltmeter and R in parallel and the voltage
resistances of ratio arms are 3 : 1, calculate the
measured after intervals of time.
distance of the fault from the test end. [Ans. 250 m]
452 Electrical and Electronic Measurements and Instrumentation

18. A telephone line, 5 km long, has an earth fault


2.3 km from test end. If the resistance of the lines
per km is 4.0 Q, what value of variable resistance
will give balance in a Varley loop test ? The ratio
arms are equal. [Ans. 21.6 fi]
19. A telephone wire having a resistance of 14.6 Q km,
develops a fault to earth. When looped with a
sound wire of the same length, the total resistance
is found to be 56 Q. If the value of variable
resistance is 16.3 at balance in a Varley loop test
with ratio arms equal, calculate the distance of
fault. [Ans. 1.36 km]
20. A Wheatstone bridge is connected for a Varley Ri = 1000Q, R2 = 100Q and Rg=52.9Q. If the
loop test as shown in Fig. 14.41. When the switch is resistance of the shorted wire is 0.015 Q/m. What is
in' position a, the bridge is balanced with the distance between the place of fault and the
Rj = 1000 0,^2 = 100 Q, Rg = 53 Q. When switch S bridge ? [Ans. 2353 m]
is in position b, the bridge is balanced with

Objective Type Questions


Tick (y) the most appropriate answer : deflection of 60° with the same setting of the
rheostat. The value of unknown resistance
1. A resistance of value 10 Q approximately is to be (a) 100 Q
measured by ammeter-voltmeter method with
(b) 54 Q
resistance of ammeter is 0.02 Q and that of
(c) 90 Q
voltmeter is 5000 Q. The resistance should be
(d) none of the above.
measured
(a) by connecting the ammeter on the side of 3. Equal resistances of 100 Q each are connected in
unknown resistance as this connection gives each arm of a Wheatstone bridge which is supplied
better accuracy by a 2 V battery source. The galvanometer of
negligible resistance connected to the bridge can
(b) by connecting the voltmeter on the side of
unknown resistance as this connection gives sense as low current as 1 g A. The smallest value of
better accuracy resistance that can be measured is :

(c) by any of the two connections, as both of them (fl) 20 gQ


give equal accuracy (b) 2 gQ
(d) none of the above. (c) 20 gQ

2. A unknown resistance is measured by substitution (d) none of the above.


method. First a standard known resistance of 100 Q 4. A Wheatstone bridge has ratio arms of 1000 Q and
is connected in series with a circuit having a 100 Q resistance, the standard resistance arms
rheostat and a galvanometer. The battery voltage is consists 4 decade resistance boxes of 1000,100,10,
10 V and the setting of the rheostat is 500 Q and the 1 Q steps. The maximum and minimum values of
galvanometer shows a deflection of 60°. After this unknown resistance which can be determined with
test, the battery voltage goes down to 9 V and this set up is
when the unknown resistance is substituted for the
(fl) 111100 Q,1Q
known resistance, the galvanometer again shows a
Measurement of Resistance 453

(b) 11110Q, 10 Q (b) guard the resistance against stray electrostatic


(c) 111100Q, 10 Q fields
(d) none of the above. (c) guard the resistance against overloads
5. A Wheatstone bridge cannot be used for precision (d) none of the above.
measurements because errors are introduced into
10. When measuring insulation resistance of cables
on account of
using d.c. source, the galvanometer used should be
(a) resistance of connecting leads initially short circuited because,
(b) thermo-electric emfs
(a) cables have a low value of initial resistance
(c) contact resistances
(b) cables have a high value of capacitance which
(d) all the above. draws a high value of charging current

6. A Kelvin-Varley Slide consists of 4 decade (c) cables have a low value of capacitance which
dividers. The first decade is constructed by having draws a high value of charging current
11 coils of 10 kQ resistance each. The subsequent (d) none of the above.
decades will have coils of :
11. A circular piece of specimen has a surface
(fl) 11 coils of 20 kQ each, 11 coils of 40 kQ each, 11 resistance Rg. Its diameter is d and the thickness is
coils of 80 kQ each
t. The surface resistivity ps of the specimen is given
(b) 11 coils of 10 kQ each, 11 coils of 5 kQ each, 10 by:
coils of 1 kQ each
7td2Ks
(c) 11 coils of 2 kQ each, 11 coils of 400 Q each, 11 («)
t
coils of 80 Q each
*dRs
(d) 11 coils of 2 kQ each, 11 coils of 400 Q each, 10
t
coils of 80 Q each.

7. Low resistances are provided with four terminals (c)


nd2
(a) to facilitate the connection of current and
potential circuits (d)
nd
(b) in order that the resistance value becomes defi­
12. The value of resistance of an earthing electrode
nite irrespective of the nature of contacts at the
depends upon :
current terminals
(c) to eliminate the effect of thermo-electric emfs (a) shape and material of electrode

(d) to eliminate the effect of leads. (b) depth to which electrode is driven into earth.

8. In a Kelvin's Double Bridge two sets of readings (c) specific resistance of soil
are taken when measuring a low resistance, one (d) all the above.
with the current in one direction and the other with 13. From the point of view of safety, the resistance of
direction of current reversed. earthing electrode should be :
This is done to (a) low
(a) eliminate the effect of contact resistance (b) high
(b) eliminate the effect of resistance of leads (c) medium
(c) correct for changes in battery voltage (d) the value of resistance of earth electrodes does
(d) eliminate the effect of thermo-electric emfs. not affect the safety.
14. The advantage of Varley loop tests over Murray
9. High resistances are provided with a guard
loop tests is
terminal.
(a) they can be used for localizing of short circuit
This guard terminal is used to :
fauits
(fl) bypass the leakage current
(b) they can be used for localizing of earth fault
454 Electrical and Electronic Measurements and Instrumentation

(c) the loop resistance can be experimentally deter­ (a) affects the balance conditions
mined
(fe) affects the value of cable resistance
(d) their accuracy is higher.
(c) affects the sensitivity of the bridge
15. When localizing ground fault with the help of loop
(d) all the above.
tests, the resistance of the fault:

\ 5* \ 3 ’C sf ? M •
l.(c) X(t) 3. (a) 4. (c) (S.(d) 6- (rf) 7. (fe) 8. (d) 9. (a) 10. (fe)
11. (!>) 12. W 13. (a) 14. (c) 15. (c)

You might also like